5

積分botを解けるだけ解く, その2

521
1
$$$$

前回( https://mathlog.info/articles/1876 ) に引き続き, 積分botの式を証明していきたいと思います.

11個目( https://twitter.com/integralsbot/status/1361174371876802560 )

$$\begin{eqnarray*} \int_0^{\infty}\frac{\sin\pi x^2\cosh\pi x}{\sinh^2 \pi x}\,dx=\frac 14 \end{eqnarray*}$$

こんな積分ができるのか, と思いました. 美しいですね. 思いつくまでが長かったです.

実数$a$に対し, 主値積分
$$\begin{eqnarray*} I(a):=\int_{-\infty}^{\infty}\frac{e^{i\pi x^2}}{\sinh\pi(x+a)}\,dx \end{eqnarray*}$$
を考える. $\frac 1{\sinh \pi (x+a)}$$0\leq\Im(a)\leq\frac{1}{2}$において, $\Re(x)\to\pm\infty$で一様に$0$に収束するので, 留数定理より, ちょうど$-a$における留数の半分が入っているとみなせることを考えて,
$$\begin{eqnarray*} I(a)&=&\int_{-\infty}^{\infty}\frac{e^{i\pi (x+i/2)^2}}{\sinh\pi\left(x+\frac i2+a\right)}\,dx+\pi iRes_{x=-a}\frac{e^{i\pi x^2}}{\sinh\pi(x+a)}\\ &=&-ie^{-i\pi/4}\int_{-\infty}^{\infty}\frac{e^{i\pi x^2-\pi x}}{\cosh\pi(x+a)}\,dx+ie^{i\pi a^2} \end{eqnarray*}$$
同様に,
$$\begin{eqnarray*} I(a)&=&\int_{-\infty}^{\infty}\frac{e^{i\pi (x-i/2)^2}}{\sinh\pi\left(x-\frac i2+a\right)}\,dx-\pi iRes_{x=-a}\frac{e^{i\pi x^2}}{\sinh\pi(x+a)}\\ &=&ie^{-i\pi/4}\int_{-\infty}^{\infty}\frac{e^{i\pi x^2+\pi x}}{\cosh\pi(x+a)}\,dx-ie^{i\pi a^2} \end{eqnarray*}$$
よって,
$$\begin{eqnarray*} \int_{-\infty}^{\infty}\frac{e^{i\pi x^2+\pi x}}{\cosh\pi(x+a)}\,dx&=&-ie^{i\pi/4}I(a)+e^{i\pi a^2+i\pi/4}\\ \int_{-\infty}^{\infty}\frac{e^{i\pi x^2-\pi x}}{\cosh\pi(x+a)}\,dx&=&ie^{i\pi/4}I(a)+e^{i\pi a^2+i\pi/4} \end{eqnarray*}$$
これより,
$$\begin{eqnarray*} e^{i\pi/4}&=&\int_{-\infty}^{\infty}e^{i\pi x^2}\,dx\\ &=&\frac 12\left(e^{\pi a}\int_{-\infty}^{\infty}\frac{e^{i\pi x^2+\pi x}}{\cosh\pi(x+a)}\,dx+e^{-\pi a}\int_{-\infty}^{\infty}\frac{e^{i\pi x^2-\pi x}}{\cosh\pi(x+a)}\,dx\right)\\ &=&\frac 12\left(-ie^{i\pi/4+\pi a}I(a)+e^{i\pi a^2+i\pi/4+\pi a}+ie^{i\pi/4-\pi a}I(a)+e^{i\pi a^2+i\pi/4-\pi a}\right)\\ &=&e^{i\pi/4}\left(-iI(a)\sinh\pi a+e^{i\pi a^2}\cosh\pi a\right) \end{eqnarray*}$$
よって,
$$\begin{eqnarray*} I(a)=\frac{i(1-e^{i\pi a^2}\cosh\pi a)}{\sinh\pi a} \end{eqnarray*}$$
虚部を考えることにより, 主値積分,

$$\begin{eqnarray*} \int_{-\infty}^{\infty}\frac{\sin\pi x^2}{\sinh(x+a)}\,dx&=&\frac{1-\cos\pi a^2\cosh\pi a}{\sinh\pi a} \end{eqnarray*}$$
を得て, 両辺は$a\to 0$において,
$$\begin{eqnarray*} \int_{-\infty}^{\infty}\sin\pi x^2\left(\frac 1{\sinh \pi x}-\frac{\pi a\cosh\pi x}{\sinh^2\pi x}+O(a^2)\right)\,dx&=&-\frac{\pi}{2}a+O(a^2) \end{eqnarray*}$$
より,
$$\begin{eqnarray*} \int_{-\infty}^{\infty}\frac{\sin\pi x^2\cosh\pi x}{\sinh^2\pi x}\,dx&=&\frac 12 \end{eqnarray*}$$
よって,
$$\begin{eqnarray*} \int_{0}^{\infty}\frac{\sin\pi x^2\cosh\pi x}{\sinh^2\pi x}\,dx&=&\frac 14 \end{eqnarray*}$$
が示された.

12個目( https://twitter.com/integralsbot/status/1361181130934771714 )

$$\begin{eqnarray*} \int_0^1\frac{x^2\ln\frac 1x}{(1-x^4)\left(\pi^2+\left(\ln\frac{1+x}{1-x}+2\tan^{-1}\frac 1x\right)^2\right)}\,dx=\frac 1{32}\ln\frac{\pi^2}{8} \end{eqnarray*}$$

見た目がすごい複雑ですね. 前の記事( https://mathlog.info/articles/1876 ) の命題9に帰着させることができました. その綺麗な証明は前回の記事のコメントで教えていただきました.

$$\begin{eqnarray*} &&\int_0^1\frac{x^2\ln\frac 1x}{(1-x^4)\left(\pi^2+\left(\ln\frac{1+x}{1-x}+2\tan^{-1}\frac 1x\right)^2\right)}\,dx\\ &=&-\int_0^1\frac{x^2\ln x}{(1-x^4)\left(\pi^2+\left(\pi+2\tanh^{-1}x-2\tan^{-1}x\right)^2\right)}\,dx\\ &=&-\frac 14\int_0^1\frac{x^2\ln x}{(1-x^4)\left(\left(\frac{\pi}{2}\right)^2+\left(\frac{\pi}2+\tanh^{-1}x-\tan^{-1}x\right)^2\right)}\,dx\\ &=&-\frac 1{2\pi}\Im\left(\int_0^1\frac{x^2\ln x}{(1-x^4)\left(\frac{\pi}{2}(1-i)+\tanh^{-1}x-\tan^{-1}x\right)}\,dx\right)\\ &=&-\frac 1{2\pi}\Im\left((1+i)\int_0^1\frac{x^2\ln x}{(1-x^4)\left(\pi+(1+i)(\tanh^{-1}x-\tan^{-1}x)\right)}\,dx\right)\\ &=&-\frac 1{4\pi}\Im\left(\left[\ln(\pi+(1+i)(\tanh^{-1}x-\tan^{-1}x))\ln x\right]_0^1\right)\\ &+&\frac 1{4\pi}\Im\left(\int_0^1\frac{\ln(\pi+(1+i)(\tanh^{-1}x-\tan^{-1}x))}{x}\,dx\right)\\ &=&\frac 1{4\pi}\int_0^1\tan^{-1}\left(\frac{\tanh^{-1}x-\tan^{-1}x}{\pi+\tanh^{-1}-\tan^{-1}x}\right)\,\frac{dx}x\\ &=&\frac{1}{32}\ln\frac{\pi^2}{8} \end{eqnarray*}$$

13個目( https://twitter.com/integralsbot/status/1361184328609374208 )

$$\begin{eqnarray*} &&\int_0^{\pi/2}\arctan\frac{2r\sin x}{1-r^2}\,dx=2\chi_2(r)\\ &&\int_0^{\pi/2}\arctan\frac{2r\sin x}{1-r^2}\arctan\frac{2s\sin x}{1-s^2}\,dx=\pi\chi_2(rs) \end{eqnarray*}$$

ここで, $\chi_2(r)$はLegendreのカイ関数といい,
$$\begin{eqnarray*} \chi_2(r):=\sum_{0\leq n}\frac{r^{2n+1}}{(2n+1)^2}, \quad(|r|\leq 1) \end{eqnarray*}$$
で定義されます. 見た目難しそうに見えるかもしれませんが, あることに気づけば簡単です.

まず,
$$\begin{eqnarray*} \arctan\frac{2r\sin x}{1-r^2}&=&\Im(\ln(1-r^2+2ir\sin x))\\ &=&\Im(\ln((1-re^{-ix})(1+re^{ix})))\\ &=&\Im(\ln(1-re^{-ix})+\ln(1+re^{ix}))\\ &=&\Im\left(-\sum_{0\lt n}\frac{r^n}{n}e^{-inx}+\sum_{0\lt n}\frac{(-1)^{n-1}r^n}{n}e^{inx}\right))\\ &=&\sum_{0\lt n}\frac{r^n}{n}\sin nx+\sum_{0\lt n}\frac{(-1)^{n-1}r^n}{n}\sin nx\\ &=&2\sum_{0\lt n}\frac{r^{2n+1}}{2n+1}\sin(2n+1)x \end{eqnarray*}$$
よって,
$$\begin{eqnarray*} \int_0^{\pi/2}\arctan\frac{2r\sin x}{1-r^2}\,dx&=&2\sum_{0\leq n}\int_0^{\pi/2}\frac{r^{2n+1}}{2n+1}\sin(2n+1)x\,dx\\ &=&2\sum_{0\leq n}\frac{r^{2n+1}}{(2n+1)^2}\\ &=&2\chi_2(r) \end{eqnarray*}$$
また, 三角関数の直交性より,
$$\begin{eqnarray*} &&\int_0^{\pi/2}\arctan\frac{2r\sin x}{1-r^2}\arctan\frac{2s\sin x}{1-s^2}\,dx\\ &=&\frac 14\int_0^{2\pi}\arctan\frac{2r\sin x}{1-r^2}\arctan\frac{2s\sin x}{1-s^2}\,dx\\ &=&\sum_{0\leq n,m}\int_0^{2\pi}\frac{r^{2n+1}s^{2m+1}}{(2n+1)(2m+1)}\sin(2n+1)x\sin(2m+1)x\,dx\\ &=&\pi\sum_{0\leq n,m}\frac{r^{2n+1}s^{2m+1}}{(2n+1)(2m+1)}\delta_{n,m}\\ &=&\pi\sum_{0\leq n}\frac{(rs)^{2n+1}}{(2n+1)^2}\\ &=&\pi\chi_2(rs) \end{eqnarray*}$$
ここで, $\delta_{n,m}$はKroneckerのデルタである.

14個目( https://twitter.com/integralsbot/status/1361197255168978945 )

$$\begin{eqnarray*} \int_0^1\frac{dx}{\sqrt[4]{x}\sqrt[4]{1-x}\sqrt{1-x\left(1-\tan\frac{3\pi}{20}\right)^2}}=\frac{\sqrt{2}}{5}\frac{\pi}{1-\tan\frac{3\pi}{20}} \end{eqnarray*}$$

超幾何積分ですね, とりあえず$\alpha=1-\tan\frac{3\pi}{20}$とおいてみると,

$$\begin{eqnarray*} \frac{\Gamma(c)}{\Gamma(b)\Gamma(c-b)}\int_0^1x^{b-1}(1-x)^{c-b-1}(1-zx)^{-a}\,dx={}_2F_1\left[\begin{matrix}a,b\\c\end{matrix};z\right] \end{eqnarray*}$$
を用いて,
$$\begin{eqnarray*} \int_0^1\frac{dx}{\sqrt[4]{x}\sqrt[4]{1-x}\sqrt{1-\alpha^2x}}&=&\frac{\Gamma\left(\frac 34\right)^2}{\Gamma\left(\frac 32\right)}{}_2F_1\left[\begin{matrix}\frac 12,\frac 34\\\frac 32\end{matrix};\alpha^2\right]\\ &=&\frac{2\Gamma\left(\frac 34\right)^2}{\alpha\sqrt{\pi}}\int_0^{\alpha}(1-x^2)^{-3/4}\,dx \end{eqnarray*}$$
となります. よって,
$$\begin{eqnarray*} \int_0^{\alpha}(1-x^2)^{-3/4}\,dx=\frac{\pi}{5\sqrt{2}}\frac{\sqrt{\pi}}{\Gamma\left(\frac 34\right)^2} \end{eqnarray*}$$
を示せばいいわけですが, 全くわかりません, $1-\tan\frac{3\pi}{20}$に固有の置換でもあるのでしょうか.

15個目( https://twitter.com/integralsbot/status/1361198231703674881 )

$$\begin{eqnarray*} \int_0^1\left(\left\lfloor\frac{\alpha}{x}\right\rfloor -\alpha\left\lfloor\frac 1x\right\rfloor\right)\,dx=\alpha\ln\alpha\quad(0\lt \alpha\lt 1) \end{eqnarray*}$$

これは最初に積分区間を$[1/N,1]$としておいてから, $N\to\infty$とする方針でいきたいと思います.

$$\begin{eqnarray*} &&\int_{1/N}^1\left(\left\lfloor\frac{\alpha}{x}\right\rfloor -\alpha\left\lfloor\frac 1x\right\rfloor\right)\,dx\\ &=&\int_{1/N}^1\left\lfloor\frac{\alpha}{x}\right\rfloor\,dx -\alpha\int_{1/N}^1\left\lfloor\frac 1x\right\rfloor\,dx\\ &=&\alpha\int_{\alpha}^{\alpha N}\frac{\lfloor x\rfloor}{x^2}\,dx-\alpha\int_1^N\frac{\lfloor x\rfloor}{x^2}\,dx\\ &=&\alpha\int_N^{\alpha N}\frac{\lfloor x\rfloor}{x^2}\,dx\\ &=&\alpha\int_N^{\alpha N}\frac 1x\,dx-\alpha\int_N^{\alpha N}\frac{x-\lfloor x\rfloor}{x^2}\,dx\\ &=&\alpha\ln\alpha-\alpha\int_N^{\alpha N}\frac{x-\lfloor x\rfloor}{x^2}\,dx \end{eqnarray*}$$
ここで, 第2項の積分は
$$\begin{eqnarray*} \int_1^{\infty}\frac{x-\lfloor x\rfloor}{x^2}\,dx&\leq&\int_1^{\infty}\frac{1}{x^2}\,dx\\ &=&1 \end{eqnarray*}$$
が収束することにより, $N\to\infty$$0$に収束する.

16個目( https://twitter.com/integralsbot/status/1361212329208082434 )

$$\begin{eqnarray*} \int_0^{\infty}\frac{\sqrt[\phi]{x}\arctan x}{(1+x^{\phi})^2}\,dx=\frac{\pi}{4\phi} \end{eqnarray*}$$

黄金比は$\phi=\frac{1+\sqrt{5}}{2}$と定義されます. ここでは, $\frac 1{\phi}=\phi-1$であることをもちいていきます. とりあえず, $x\to \frac 1x$と置換してみるといいと思います.

$$\begin{eqnarray*} I&=&\int_0^{\infty}\frac{\sqrt[\phi]{x}\arctan x}{(1+x^{\phi})^2}\,dx\\ &=&\int_0^{\infty}\frac{x^{\phi-1}\arctan x}{(1+x^{\phi})^2}\,dx\\ &=&\int_0^{\infty}\frac{x^{-\phi-1}\arctan\frac 1x}{(1+x^{-\phi})^2}\,dx\\ &=&\int_0^{\infty}\frac{x^{\phi-1}\arctan\frac 1x}{(1+x^{\phi})^2}\,dx \end{eqnarray*}$$
よって,
$$\begin{eqnarray*} I&=&\frac 12\int_0^{\infty}\frac{x^{\phi-1}\left(\arctan x+\arctan\frac 1x\right)}{(1+x^{\phi})^2}\,dx\\ &=&\frac{\pi}{4}\int_0^{\infty}\frac{x^{\phi-1}}{(1+x^{\phi})^2}\,dx\\ &=&\frac{\pi}{4\phi}\int_0^{\infty}\frac{1}{(1+x)^2}\,dx\\ &=&\frac{\pi}{4\phi} \end{eqnarray*}$$

17個目( https://twitter.com/integralsbot/status/1361213210221764609 )

$$\begin{eqnarray*} \int_0^1\frac{dx}{\sqrt[4]{x}\sqrt[4]{1-x}\sqrt{1-x\beta^2}}=\frac 17\frac{2\pi}{\sqrt{2}|\beta|}\\ for\quad\beta^2-\frac{2}{1+2\cos\frac{\pi}{14}}\beta+\frac{4}{2-\sqrt{2}\sec\frac{13\pi}{28}}=0\\ \int_0^1\frac{dx}{\sqrt[4]{x}\sqrt{1-x}\sqrt{1-x\gamma^2}}=\frac 17\frac{2\pi}{\sqrt{2\gamma}}\\ for\quad\gamma^2-2\cos\frac{2\pi}{7}\gamma+\frac{2-\sqrt{2}\csc\frac{9\pi}{28}}{4}=0\\ \end{eqnarray*}$$

こ, これは, さっきの命題4と同じ関数, と思いきや2つ目の積分は微妙に違いますね. とりあえず, わかる気がしないので次いきましょう.

18個目( https://twitter.com/integralsbot/status/1361219897196376065 )

$$\begin{eqnarray*} \int_0^{\pi/2}\frac{\arctan\sqrt{\tan x}}{\tan x}\,dx=\frac{\pi}{2}\ln(2+\sqrt{2}) \end{eqnarray*}$$

さっそく証明していきます.

置換$\tan x\to x^2$により,
$$\begin{eqnarray*} \int_0^{\pi/2}\frac{\arctan\sqrt{\tan x}}{\tan x}\,dx&=&2\int_0^{\infty}\frac{\arctan x}{x(1+x^4)}\,dx\\ &=&\int_{-\infty}^{\infty}\frac{\arctan x}{x(1+x^4)}\,dx \end{eqnarray*}$$
ここで, $0\lt\Im(a)$として,
$$\begin{eqnarray*} \int_{-\infty}^{\infty}\frac{1}{(x-a)(1+b^2x^2)}\,dx&=&\frac 1{1+a^2b^2}\int_{-\infty}^{\infty}\left(\frac{1}{x-a}-\frac{b^2(x+a)}{1+b^2x^2}\right)\,dx\\ &=&\frac 1{1+a^2b^2}\lim_{A\to-\infty,B\to\infty}\left[\ln(x-a)-\frac 12\ln(1+b^2x^2)-ab\arctan bx\right]_A^B\\ &=&\frac 1{1+a^2b^2}\pi(i-ab)\\ &=&-\frac{\pi}{i+ab} \end{eqnarray*}$$
よって,
$$\begin{eqnarray*} \int_{-\infty}^{\infty}\frac{\arctan bx}{x(x-a)}\,dx&=&\frac{\pi}{a}\ln(1-iab)+C(a) \end{eqnarray*}$$
となる$a$に依存する関数があるが, $b=0$として, $C(a)=0$である. 両辺の共役を考えることにより, $a$を実数でない複素数としたとき,
$$\begin{eqnarray*} \int_{-\infty}^{\infty}\frac{\arctan bx}{x(x-a)}\,dx&=&\begin{cases} -\frac{\pi}{a}\ln(1-iab)\quad(0\lt\Im(a))\\ -\frac{\pi}{a}\ln(1+iab)\quad(0\gt\Im(a))\\ \end{cases} \end{eqnarray*}$$
よって,
$$\begin{eqnarray*} \int_{-\infty}^{\infty}\frac{\arctan x}{x(1+x^4)}\,dx&=&-\frac 14\int_{-\infty}^{\infty}\frac{\arctan x}{x}\left(\frac{e^{i\pi/4}}{x-e^{i\pi/4}}+\frac{e^{-i\pi/4}}{x-e^{-i\pi/4}}+\frac{e^{3i\pi/4}}{x-e^{3i\pi/4}}+\frac{e^{-3i\pi/4}}{x-e^{-3i\pi/4}}\right)\,dx\\ &=&\frac{\pi}4\left(\ln(1-ie^{i\pi/4})+\ln(1+ie^{-i\pi/4})+\ln(1-ie^{3i\pi/4})+\ln(1+ie^{3i\pi/4})\right)\\ &=&\frac{\pi}{2}\ln(2+\sqrt{2}) \end{eqnarray*}$$

19個目( https://twitter.com/integralsbot/status/1361230834020327430 )

$$\begin{eqnarray*} \int_0^1\frac{x}{(1-s^2x^2)\sqrt{1-x^2}}\ln\frac{1-x}{1+x}\,dx=\frac{\pi}{2s\sqrt{1-s^2}}\ln\frac{1-s}{1+s} \end{eqnarray*}$$

これはかなり難しかったです. もっと綺麗な方法があるかもしれません.

まず, 与えられた式は,
$$\begin{eqnarray*} \int_0^1\frac{x\tanh^{-1}x}{(1-s^2x^2)\sqrt{1-x^2}}\,dx=\frac{\pi\tanh^{-1}s}{2s\sqrt{1-s^2}} \end{eqnarray*}$$
と書き直せるので, これを示します.
$$\begin{eqnarray*} &&\int_0^1\frac{x\tanh^{-1}x}{(1-s^2x^2)\sqrt{1-x^2}}\,dx\\ &=&\sum_{0\leq n}\frac 1{2n+1}\int_0^1 x^{2n+2}(1-x^2)^{-1/2}(1-s^2x^2)^{-1}\,dx\\ &=&\sum_{0\leq n,m}\frac {s^{2m}}{2n+1}\int_0^1 x^{2n+2m+2}(1-x^2)^{-1/2}\,dx\\ &=&\frac 12\sum_{0\leq n,m}\frac{s^{2m}}{2n+1}\int_0^1x^{n+m+1/2}(1-x)^{-1/2}\,dx\\ &=&\frac 12\sum_{0\leq n,m}\frac{s^{2m}}{2n+1}\frac{\Gamma\left(n+m+\frac 32\right)\Gamma\left(\frac 12\right)}{\Gamma(n+m+2)}\\ &=&\frac{\pi}2\sum_{0\leq n,m}\frac{\left(\frac 12\right)_{n+m+1}}{(n+m+1)!(2n+1)}s^{2m}\\ &=&\frac{\pi}{2}\sum_{0\leq m}s^{2m}\sum_{n=m+1}^{\infty}\frac{\left(\frac 12\right)_n}{n!(2n-2m-1)} \end{eqnarray*}$$
一方,
$$\begin{eqnarray*} \frac{\pi\tanh^{-1}s}{2s\sqrt{1-s^2}}&=&\frac{\pi}{2}\sum_{0\leq n,m}\frac{\left(\frac 12\right)_n}{n!(2m+1)}s^{2n+2m}\\ &=&\frac{\pi}{2}\sum_{0\leq m}s^{2m}\sum_{n=0}^m\frac{\left(\frac 12\right)_n}{n!(2m-2n+1)} \end{eqnarray*}$$
より,
$$\begin{eqnarray*} \sum_{n=m+1}^{\infty}\frac{\left(\frac 12\right)_n}{n!(2n-2m-1)}=\sum_{n=0}^m\frac{\left(\frac 12\right)_n}{n!(2m-2n+1)} \end{eqnarray*}$$
を示せばよいが, これはGaussの超幾何定理より,
$$\begin{eqnarray*} &&\sum_{n=m+1}^{\infty}\frac{\left(\frac 12\right)_n}{n!(2n-2m-1)}-\sum_{n=0}^m\frac{\left(\frac 12\right)_n}{n!(2m-2n+1)}\\ &=&\sum_{n=0}^{\infty}\frac{\left(\frac 12\right)_n}{n!(2n-2m-1)}\\ &=&-\frac{2}{2m+1}\,{}_2F_1\left[\begin{matrix}\frac 12,-\frac 12-m\\\frac 12-m\end{matrix};1\right]\\ &=&0 \end{eqnarray*}$$
より従う.

20個目( https://twitter.com/integralsbot/status/1361242196796715014 )

$$\begin{eqnarray*} \int_0^{\infty}J_n(bx)x^ne^{-ax}\,dx=\frac{(2b)^n\Gamma\left(\frac 12+n\right)}{\sqrt{\pi}(a^2+b^2)^{1/2+n}} \end{eqnarray*}$$

Bessel関数の性質はよく知らないので, とりあえず項別積分ですね. 収束範囲とか知らないけど, $|b|\geq |a|$のときは解析接続で示せるのかな.

$|b|\lt |a|$とする.
$$\begin{eqnarray*} \int_0^{\infty}J_n(bx)x^ne^{-ax}\,dx&=&\frac{2^n}{b^n}\sum_{0\leq m}\frac{(-1)^m}{m!(n+m)!}\int_0^{\infty}\left(\frac{bx}2\right)^{2m+2n}e^{-ax}\,dx\\ &=&\frac{2^n}{ab^n}\sum_{0\leq m}\frac{(-1)^m(2n+2m)!}{m!(n+m)!}\left(\frac{b}{2a}\right)^{2n+2m}\\ &=&\frac{(2b)^n}{a^{2n+1}}\sum_{0\leq m}\frac{(-1)^m\left(\frac 12\right)_{n+m}}{m!}\left(\frac{b}{a}\right)^{2m}\\ &=&\frac{(2b)^n\left(\frac 12\right)_n}{a^{2n+1}}\sum_{0\leq m}\frac{\left(\frac 12+n\right)_n}{m!}\left(-\frac{b^2}{a^2}\right)^m\\ &=&\frac{(2b)^n\Gamma\left(\frac 12+n\right)}{\sqrt{\pi}a^{2n+1}}\left(1+\frac{b^2}{a^2}\right)^{-1/2-n}\\ &=&\frac{(2b)^n\Gamma\left(\frac 12+n\right)}{\sqrt{\pi}(a^2+b^2)^{1/2+n}} \end{eqnarray*}$$

今のところ, 10問中2問ぐらいは全く分からないのが混ざってる感じがします. やっていくうちにもっと多かったり少なかったりするかもしれませんが.

投稿日:202137

この記事を高評価した人

高評価したユーザはいません

この記事に送られたバッジ

バッジはありません。

投稿者

Wataru
Wataru
404
25179
超幾何関数, 直交関数, 多重ゼータ値などに興味があります

コメント

他の人のコメント

コメントはありません。
読み込み中...
読み込み中